LSAT and Law School Admissions Forum

Get expert LSAT preparation and law school admissions advice from PowerScore Test Preparation.

User avatar
 Dave Killoran
PowerScore Staff
  • PowerScore Staff
  • Posts: 5862
  • Joined: Mar 25, 2011
|
#41582
Complete Question Explanation
(The complete setup for this game can be found here: lsat/viewtopic.php?t=11733)

The correct answer choice is (E)

The question stem produces the following setup:
J91_Game_#4_#19_diagram 1.png
If the line 3 tickets for January are R, then from the fifth rule the line 3 tickets for February must be G. Hence, answer choice (E) is correct.
You do not have the required permissions to view the files attached to this post.
 tthauvette
  • Posts: 7
  • Joined: Aug 18, 2016
|
#29963
This question is about the railway tickets on p4-44 of Lesson 4 homework.
19. If the line 3 tickets for January are red, then which one of the following statements must be true.

I narrowed down:
b) The line 1 tickets for January are yellow
e) The line 3 tickets for February are green

why is one more true than the other? Can you explain please.

thanks,

Tania
 David Boyle
PowerScore Staff
  • PowerScore Staff
  • Posts: 836
  • Joined: Jun 07, 2013
|
#30055
tthauvette wrote:This question is about the railway tickets on p4-44 of Lesson 4 homework.
19. If the line 3 tickets for January are red, then which one of the following statements must be true.

I narrowed down:
b) The line 1 tickets for January are yellow
e) The line 3 tickets for February are green

why is one more true than the other? Can you explain please.

thanks,

Tania

Hello Tania,

Answer B is not true because January line 1 could also be green, not necessarily yellow.

Hope this helps,
David

Get the most out of your LSAT Prep Plus subscription.

Analyze and track your performance with our Testing and Analytics Package.